mirror of
https://github.com/MartinThoma/LaTeX-examples.git
synced 2025-04-19 11:38:05 +02:00
added images; added solution
This commit is contained in:
parent
cbe61b4ea1
commit
0e005dabca
6 changed files with 56 additions and 6 deletions
Binary file not shown.
|
@ -34,6 +34,7 @@
|
|||
\usepackage{tikz-3dplot} % draw
|
||||
\usepackage{tkz-fct} % draw
|
||||
\usetikzlibrary{3d,calc,intersections,er,arrows,positioning,shapes.misc,patterns,fadings,decorations.pathreplacing}
|
||||
\usepackage{tqft}
|
||||
\usepackage{cleveref} % has to be after hyperref, ntheorem, amsthm
|
||||
\usepackage{shortcuts}
|
||||
|
||||
|
|
|
@ -19,7 +19,6 @@
|
|||
Zeigen Sie:
|
||||
\begin{enumerate}[label=(\alph*)]
|
||||
\item Jedes $U_{a,b}$ und jede einelementige Teilmenge von $\mdz$ ist abgeschlossen.
|
||||
\item Die $U_{a,b}$ bilden eine Basis der Topologie.
|
||||
\item $\Set{-1, 1}$ ist nicht offen.
|
||||
\item Es gibt unendlich viele Primzahlen.
|
||||
\end{enumerate}
|
||||
|
|
|
@ -298,7 +298,7 @@ Auch gibt es Mengen, die sowohl abgeschlossen als auch offen sind.
|
|||
$\Rightarrow$ Widerspruch $\qed$
|
||||
\end{beweis}
|
||||
|
||||
\section{Stetigkeit}
|
||||
\section{Stetigkeit}\index{Stetigkeit|(}
|
||||
\begin{definition} \xindex{stetig} \xindex{Homöomorphismus}
|
||||
Seien $X, Y$ topologische Räume und $f:X \rightarrow Y$ eine Abbildung.
|
||||
|
||||
|
@ -470,6 +470,7 @@ sodass $\pi$ stetig wird.
|
|||
%%%%%%%%%%%%%%%%%%%%%%%%%%%%%%%%%%%%%%%%%%%%%%%%%%%%%%%%%%%%%%%%%%%%%
|
||||
% Mitschrieb vom 31.10.2013 %
|
||||
%%%%%%%%%%%%%%%%%%%%%%%%%%%%%%%%%%%%%%%%%%%%%%%%%%%%%%%%%%%%%%%%%%%%%
|
||||
\index{Stetigkeit|)}
|
||||
\section{Zusammenhang}\index{Zusammenhang|(}
|
||||
\begin{definition}\xindex{zusammenhängend}
|
||||
Ein Raum $X$ heißt \textbf{zusammenhängend}, wenn es keine offenen
|
||||
|
@ -799,7 +800,7 @@ $\qed$
|
|||
%%%%%%%%%%%%%%%%%%%%%%%%%%%%%%%%%%%%%%%%%%%%%%%%%%%%%%%%%%%%%%%%%%%%%
|
||||
% Mitschrieb vom 07.11.2013 %
|
||||
%%%%%%%%%%%%%%%%%%%%%%%%%%%%%%%%%%%%%%%%%%%%%%%%%%%%%%%%%%%%%%%%%%%%%
|
||||
\section{Wege und Knoten}
|
||||
\section{Wege und Knoten}\index{Knoten|(}
|
||||
\begin{definition}\xindex{Weg}\xindex{Weg!geschlossener}\xindex{Weg!einfacher}
|
||||
Sei $X$ ein topologischer Raum.
|
||||
\begin{enumerate}[label=\alph*)]
|
||||
|
@ -1003,6 +1004,7 @@ $\qed$
|
|||
\label{fig:reidemeister-zuege}
|
||||
\caption{Ein 3-gefärber Kleeblattknoten}
|
||||
\end{figure}
|
||||
\index{Knoten|)}
|
||||
|
||||
% Die Übungsaufgaben sollen ganz am Ende des Kapitels sein.
|
||||
\input{Kapitel1-UB}
|
||||
|
|
|
@ -210,7 +210,7 @@ U_i = \Set{(x_0: \dots : x_n) \in \mdp^n(\mdr) | x_i \neq 0} &\rightarrow \mdr^n
|
|||
\end{enumerate}
|
||||
\end{beispiel}
|
||||
|
||||
\begin{definition}\textbf{Mannigfaltigkeit!mit Rand}
|
||||
\begin{definition}\xindex{Mannigfaltigkeit!mit Rand}
|
||||
Sei $X$ ein Hausdorffraum mit abzählbarer Basis der Topologie.
|
||||
$X$ heißt $n$-dimensionale \textbf{Mannigfaltigkeit mit Rand},
|
||||
wenn es einen Atlas $(U_i, \varphi_i)$ gibt, wobei $U_i \subseteq X_i$
|
||||
|
@ -221,7 +221,24 @@ U_i = \Set{(x_0: \dots : x_n) \in \mdp^n(\mdr) | x_i \neq 0} &\rightarrow \mdr^n
|
|||
\end{definition}
|
||||
|
||||
\begin{beispiel}
|
||||
\todo[inline]{Viele Bilder: Pair of pants, sphere with a hole, halbraum...}
|
||||
\begin{figure}[ht]
|
||||
\centering
|
||||
\subfloat[Halbraum]{
|
||||
\input{figures/topology-halfspace.tex}
|
||||
\label{fig:half-space}
|
||||
}%
|
||||
|
||||
\subfloat[Pair of pants]{
|
||||
\input{figures/topology-pair-of-pants.tex}
|
||||
\label{fig:pair-of-pants}
|
||||
}%
|
||||
\subfloat[Sphäre mit einem Loch]{
|
||||
\input{figures/topology-sphere-with-hole.tex}
|
||||
\label{fig:sphere-with-hole}
|
||||
}%
|
||||
\label{Mannigfaltigkeiten mit Rand}
|
||||
\caption{Beispiele für Mannigfaltigkeiten mit Rand}
|
||||
\end{figure}
|
||||
\end{beispiel}
|
||||
|
||||
\begin{definition}\xindex{Rand}
|
||||
|
|
|
@ -25,7 +25,38 @@
|
|||
\end{solution}
|
||||
|
||||
\begin{solution}[\ref{ub1:aufg4}]
|
||||
\todo[inline]{Lösung schreiben}
|
||||
\textbf{Teilaufgabe a)}
|
||||
|
||||
\textbf{Beh.:} $\forall a \in \mdz: \Set{a}$ ist abgeschlossen.
|
||||
|
||||
Sei $a \in \mdz$ beliebig. Dann gilt:
|
||||
\todo[inline]{Hat jemand diesen Beweis?}
|
||||
|
||||
\textbf{Teilaufgabe b)}
|
||||
|
||||
\textbf{Beh.:} $\Set{-1, 1}$ ist nicht offen
|
||||
|
||||
\textbf{Bew.:} durch Widerspruch
|
||||
|
||||
Annahme: $\Set{-1, 1}$ ist offen.
|
||||
|
||||
Dann gibt es $T \subseteq \fB$, sodass $\bigcup_{M \in T} M = \Set{-1, 1}$.
|
||||
Aber alle $U \in \fB$ haben unendlich viele Elemente. Auch endlich
|
||||
viele Schnitte von Elementen in $\fB$ haben unendlich viele
|
||||
Elemente $\Rightarrow$ keine endliche nicht-leere Menge kann
|
||||
in dieser Topologie offen sein $\Rightarrow \Set{-1,1}$ ist
|
||||
nicht offen. $\qed$
|
||||
|
||||
\textbf{Beh.:} Es gibt unendlich viele Primzahlen.
|
||||
|
||||
\textbf{Bew.:} durch Widerspruch
|
||||
|
||||
Annahme: Es gibt nur endlich viele Primzahlen $p \in \mdp$
|
||||
|
||||
Dann ist
|
||||
\[\mdz \setminus \Set{-1, +1} \overset{\text{FS d. Arithmetik}}= \bigcup_{p \in \mdp} U_{0,p}\]
|
||||
endlich. Das ist ein Widerspruch zu $|\mdz|$ ist unendlich und
|
||||
$|\Set{-1,1}|$ ist endlich. $\qed$
|
||||
\end{solution}
|
||||
|
||||
\begin{solution}[\ref{ub2:aufg4}]
|
||||
|
|
Loading…
Add table
Add a link
Reference in a new issue